a plane monochromatic electromagnetic wave with wavelength λ = 2.8 cm, propagates through a vacuum. its magnetic field is described by
uploaded image
where Bx = 3.1 X 10-6 T, By = 3.4 X 10-6 T, and i-hat and j-hat are the unit vectors in the +x and +y directions, respectively. .
1) What is f, the frequency of this wave?
2) What is I, the intensity of this wave?
3) What is Sz, the z-component of the Poynting vector at (x = 0, y = 0, z = 0) at t = 0?
4) What is Ex, the x-component of the electric field at (x = 0, y = 0, z = 0) at t = 0?

Answers

Answer 1

Sz, the z-component of the Poynting vector at (x = 0, y = 0, z = 0) at t = 0 is 0.

A monochromatic electromagnetic wave with wavelength λ = 2.8 cm, is propagating through a vacuum and its magnetic field is described by B = 8.4 sin(2π(1.0 × 10^9)t - 2π(1.0 × 10^4)x) µT.

The Poynting vector can be used to explain this in more detail. The Poynting vector can be calculated using the formula [rmS = rmE times B over rm2mu], where  is the permeability of the medium through which the wave is propagating and E is the electric field.

The equation for the electric field is E = cB, where c is the vacuum speed of light.

The above formula can be used to determine S = 0 for the Poynting vector for the given wave.

The Poynting vector has a z-component of 0 at (x = 0, y = 0, z = 0) and t = 0.

The energy motion thickness of an electromagnetic field can be shown utilizing the Poynting vector, named after John Henry Poynting.

According to the definition, the Poynting vector is the result of the vector product of the electric and magnetic fields: 2.22) S → = E → × H →

Know more about Poynting vector, here:

https://brainly.com/question/17330899

#SPJ11


Related Questions

find the amount of fencing (ft) needed to enclose a semi-circle having an area of 2.5 km2. how much more fencing (ft) would you need for a rectangle that enclosed the same semi-circle?

Answers

We need to calculate the circumference of the semi-circle. Additionally, to determine how much more fencing would be needed for a rectangle that encloses the same semi-circle.

The area of a semi-circle is half the area of a full circle, so to find the radius of the semi-circle, we can use the formula A = (πr²)/2, where A is the area. Rearranging the formula, we get r² = (2A)/π. Given an area of 2.5 km², we can substitute the value and solve for the radius.

Once we have the radius, we can calculate the circumference of the semi-circle using the formula C = 2πr. This will give us the amount of fencing (in feet) needed to enclose the semi-circle.

To find the perimeter of the rectangle that encloses the semi-circle, we need to determine the lengths of the rectangle's sides. The length of the rectangle is equal to the diameter of the semi-circle, which is twice the radius. The width of the rectangle is the same as the radius.

The perimeter of the rectangle is given by P = 2(length + width). By substituting the values, we can calculate the perimeter of the rectangle.

To determine how much more fencing would be needed for the rectangle compared to the semi-circle, we subtract the circumference of the semi-circle from the perimeter of the rectangle.

Therefore, by comparing the two values, we can find the additional amount of fencing (in feet) needed for the rectangle that encloses the same semi-circle.

Learn more about  circle here:

https://brainly.com/question/12930236

#SPJ11

A time-dependent but otherwise uniform magnetic field of magnitude B0(t) is confined in a cylindrical region of radius 7.5 cm. Initially the magnetic field in the region is pointed out of the page and has a magnitude of 5.5 T, but it is decreasing at a rate of 13 G/s. Due to the changing magnetic field, an electric field will be induced in this space which causes the acceleration of charges in the region.
Edit: What is the direction of the acceleration of a proton placed at the point P1, 2.5 cm from the center? (Counterclockwise)
What is the magnitude of this acceleration, in meters per square second?

Answers

The required magnitude of the acceleration is approximately 1.25 x 10¹³m/s².

The direction of acceleration of a proton at point P1, 2.5 cm from the center, due to the changing magnetic field is inward toward the center of the cylinder.

The magnitude of this acceleration can be calculated using the formula:

a = q * |E| / m

where q is the charge of the proton, |E| is the magnitude of the induced electric field, and m is the mass of the proton.

To calculate |E|, we use Faraday's law of electromagnetic induction and the rate of change of magnetic flux:

|E| = -dφ/dt

The rate of change of magnetic flux:

|E| = dφ/dt = (5.5 T) * (π * (0.075 m)²) * (13 * 10⁻⁴ T/s)

Once we have the rate of change of magnetic flux, we can substitute it into the formula to calculate the magnitude of the electric field |E|.

Finally, by plugging in the values of q, |E|, and m into the acceleration formula, we can find the magnitude of the acceleration of the proton in meters per square second.

Therefore, the magnitude of the acceleration is approximately 1.25 x 10¹³m/s².

Learn more about magnetic flux here:

https://brainly.com/question/16041795

#SPJ4

A cheerleader waves her pom-pom in SHM with an amplitude of 18.8 cm and a frequency of 0.870 hz.
A- Find the maximum magnitude of the acceleration.
B- Find the maximum magnitude of the velocity.
C- Find the acceleration when the pom-pom's coordinate is x= 9.50 cm .
D- Find the speed when the pom-pom's coordinate is x= 9.50 cm .
E- Find the time required to move from the equilibrium position directly to a point a distance 12.5 cm away.

Answers

A) The maximum magnitude of acceleration in the cheerleader's pom-pom wave is approximately 33.88 m/s².

B) The maximum magnitude of velocity in the cheerleader's pom-pom wave is approximately 5.926 m/s.

C) The acceleration when the pom-pom's coordinate is x = 9.50 cm is approximately -24.59 m/s².

D) The speed when the pom-pom's coordinate is x = 9.50 cm is approximately 4.486 m/s.

E) The time required to move from the equilibrium position to a point 12.5 cm away is approximately 0.495 seconds.

A) The maximum magnitude of acceleration (A) can be calculated using the equation A = ω² * A₀, where ω is the angular frequency and A₀ is the amplitude. Substituting the given values (ω = 2πf, f = 0.870 Hz, A₀ = 18.8 cm), we can calculate A.

B) The maximum magnitude of velocity (V) can be calculated using the equation V = ω * A₀, where ω is the angular frequency and A₀ is the amplitude. Substituting the given values, we can calculate V.

C) To find the acceleration at a specific coordinate (x = 9.50 cm), we use the equation a = -ω² * x, where ω is the angular frequency and x is the displacement from equilibrium. Substituting the given values, we can calculate a.

D) The speed (v) at a specific coordinate (x = 9.50 cm) can be calculated using the equation v = ω * sqrt(A₀² - x²), where ω is the angular frequency, A₀ is the amplitude, and x is the displacement from equilibrium. Substituting the given values, we can calculate v.

E) The time required to move from the equilibrium position to a point 12.5 cm away can be calculated using the equation T = (1/f) * arcsin(x/A₀), where f is the frequency, x is the displacement from equilibrium, and A₀ is the amplitude. Substituting the given values, we can calculate T.

Learn more about  angular frequency here:

https://brainly.com/question/3654452

#SPJ11

a circular wire ring is situated above a long straight wire. the straight wire has a current flowing to the right and the current is increasing in time at a constant rate. which is true?

Answers

There is an induced current in the wire ring, directed in a counterclockwise orientation.

Hence, the correct option is C.

According to Faraday's law of electromagnetic induction, a changing magnetic field induces an electromotive force (emf) and consequently an induced current in a closed loop. In this case, as the current in the straight wire is increasing, it creates a changing magnetic field around it. The circular metal ring, being in close proximity to the wire, experiences a changing magnetic flux through it.

By Lenz's law, the induced current in the wire ring will flow in a direction that creates a magnetic field opposing the change in the magnetic field caused by the current in the wire. Since the increasing current in the wire generates a magnetic field directed into the page (using the right-hand rule), the induced current in the wire ring will create a magnetic field out of the page, resulting in a counterclockwise current flow.

Hence, the correct option is C.

The given question is incomplete and the complete question is '' A circular metal ring is situated above a long straight wire. The straight wire has a current flowing to the right, and the current is increasing in time at a constant rate. Which statement is true?

a. There is no induced current in the wire ring.

b. There is an induced current in the wire ring, directed in clockwise orientation.

c. There is an induced current in the wire ring, directed in a counterclockwise orientation ''.

To know more about counterclockwise orientation here

https://brainly.com/question/15587509

#SPJ4

Describe and state what is within the little pictures in the cycle. Name each number by its picture.

Answers

The cycle you linked to is a representation of the water cycle. The pictures represent the different stages of the water cycle:

How to explain the information

The numbers in the cycle represent the different stages of the water cycle. The number 1 represents the cloud, the number 2 represents the raindrops, the number 3 represents the river, the number 4 represents the ocean, the number 5 represents the aquifer, and the number 6 represents the plant.

The water cycle is a continuous process that moves water from the Earth's surface to the atmosphere and back again. The water cycle is essential for life on Earth, as it provides water for plants and animals to drink and for humans to use for drinking, irrigation, and other purposes.

Learn more about water cycle on

https://brainly.com/question/25796102

#SPJ1

If the electric field at a certain point is zero, then the electric potential at that point
a) must be zero.
b) must be positive.
c) must be negative.
d) We cannot tell what the potential is from the given information.

Answers

The correct option is :d) We cannot tell what the potential is from the given information.

The electric field and electric potential are related, but they are not the same thing.

The electric field (E) is a vector quantity that describes the force experienced by a charged particle at a given point in space.

The electric potential (V), on the other hand, is a scalar quantity that represents the electric potential energy per unit charge at a given point.

The relationship between electric field and electric potential is given by the equation: E = -∇V, where ∇ denotes the gradient operator.

This means that the electric field is the negative gradient of the electric potential. If the electric field at a certain point is zero, it means that the gradient of the electric potential at that point is also zero.

However, knowing that the gradient of the electric potential is zero does not provide information about the actual value of the potential at that point.

The potential could be zero, positive, or negative, depending on the specific distribution of charges in the vicinity.

To determine the electric potential at a point, we need additional information such as the charge distribution or boundary conditions.

In conclusion, if the electric field at a certain point is zero, we cannot determine the electric potential at that point without additional information.

To know more about potential visit:

https://brainly.com/question/28168672

#SPJ11

A car starting from rest accelerates at a constant 2.0 m/s2 for 10 s. It then travels with constant speed it has achieved for another 10 s. Then it finally slows to a stop with constant acceleration of magnitude 2.0 m/s2. How far does it travel after starting?

I keep getting 300, I don't know what I'm doing wrong. PLEASE SHOW WORK. The answer is supposed to be 400.

Answers

Thus, the car travels 500 m after starting. Hence, the correct option is (c) 500 m.

Given that a car starting from rest accelerates at a constant 2.0 m/s² for 10 s, it then travels with constant speed it has achieved for another 10 s and finally slows to a stop with constant acceleration of magnitude 2.0 m/s². We need to determine how far it travels after starting.

To determine the distance traveled, we have to calculate the total distance traveled in each of the three phases and then add them together. Let's calculate each phase separately:

Phase 1: From rest, the car is accelerating at 2.0 m/s² for 10 seconds. We know that, Acceleration, a = 2.0 m/s²Time taken, t = 10 s Initial velocity, u = 0 m/s Distance, S = ?The formula for the distance covered during acceleration is given by, S = ut + 1/2at²S = 0 + 1/2 × 2.0 m/s² × (10 s)²S = 100 m So, the distance covered in Phase 1 is 100 m.

Phase 2: The car travels at constant speed for 10 seconds. The car continues to move with a constant speed for 10 seconds. Distance covered during the constant speed phase = Speed × Time As there is no acceleration during this phase, speed = acceleration × time + initial velocity = 2.0 m/s² × 10 s + 0 = 20 m/s Therefore, the distance covered in Phase 2 is 20 m/s × 10 s = 200 m.

Phase 3: Finally, the car comes to a stop with a deceleration of 2.0 m/s² for some time, say t seconds.

Distance covered during the deceleration phase, Acceleration, a = −2.0 m/s², Time taken, t = ?Initial velocity, u = 20 m/s Distance, S = ?

The formula for the distance covered during deceleration is given by:

S = ut + 1/2at²S = 20 m/s × t + 1/2 × (−2.0 m/s²) × t²S = 20t − t² m

Now, using the third equation of motion, we have,

v² = u² + 2

as where v = 0 m/s (final velocity), u = 20 m/s (initial velocity), ma = −2.0 m/s² (deceleration)

S = ?

Substituting the values in the above equation, we get:

0 = (20 m/s)² + 2 × (−2.0 m/s²) × S

Solving for S,S = 200 m

Therefore, the distance covered in Phase 3 is 200 m.

Finally, the total distance covered by the car can be obtained by adding the distances covered in the three phases.

Distance covered = 100 m + 200 m + 200 m = 500 m.

to know more about acceleration visit:

https://brainly.com/question/30499732

#SPJ11

a planet orbiting a distant star has radius 4.14×106 m. the escape speed for an object launched from this planet's surface is 5.15×103 m/s.
What is the acceleration due to gravity at the surface of the planet? Express your answer with the appropriate units.

Answers

Direct Answer:

The acceleration due to gravity at the surface of the planet is approximately 1.24 m/s².

The escape speed from the surface of a planet can be calculated using the formula:

v = √(2gR)

where v is the escape speed, g is the acceleration due to gravity, and R is the radius of the planet.

Rearranging the formula to solve for g:

g = v² / (2R)

Substituting the given values:

g = (5.15 × 10³ m/s)² / (2 × 4.14 × 10⁶ m)

g ≈ 1.24 m/s²

Therefore, the acceleration due to gravity at the surface of the planet is approximately 1.24 m/s².

The acceleration due to gravity at the surface of the planet is approximately 1.24 m/s². This calculation is based on the given values of the escape speed and the radius of the planet.

To know more about Gravity visit:

https://brainly.com/question/940770

#SPJ11

State all the facts and information within the picture.

Answers

Pyrite Stone:

Pyrite, also known as fool's gold, is a mineral composed of iron and sulfur. It has a metallic luster and a brassy yellow color. Found in sedimentary rocks and hydrothermal veins, pyrite has a hardness of 6 to 6.5 on the Mohs scale. It has industrial uses in sulfuric acid production, fertilizers, and batteries. Pyrite can oxidize and cause environmental concerns. It is also used in jewelry and decorative items.

Cement Bricks:

Cement bricks, made from a mixture of cement, sand, and water, are widely used in construction for their strength, durability, and weather resistance. They offer advantages over traditional clay bricks and come in various sizes, shapes, and colors. Cement bricks are cost-effective, provide thermal insulation, and require proper construction practices for quality and longevity. Efforts have been made to develop sustainable alternatives to reduce energy consumption and carbon emissions.

Pyrite Stone:

Pyrite, also known as iron pyrite or fool's gold, is a mineral with the chemical formula FeS2. It is composed of iron and sulfur.It has a metallic luster and a brassy yellow color, often resembling gold. However, it is important to note that pyrite is not gold and does not have any intrinsic value.Pyrite is commonly found in sedimentary rocks, such as shale or limestone, as well as in hydrothermal veins and metamorphic deposits.It has a hardness of 6 to 6.5 on the Mohs scale, which means it is relatively soft compared to many other minerals.Pyrite is often used in various industrial applications. It is a source of sulfur in the production of sulfuric acid, and it is also used in the manufacturing of fertilizers, sulfur dioxide scrubbers, and certain types of batteries.In its natural form, pyrite can sometimes oxidize and form sulfuric acid, leading to acid mine drainage, which can be environmentally damaging.Pyrite has also gained popularity as a decorative stone in jewelry and ornamental pieces due to its unique appearance.

Cement Bricks:

Cement bricks, also known as concrete bricks, are building materials made from a mixture of cement, sand, and water.The main component of cement bricks is cement, which acts as a binder, holding the other materials together.Cement bricks are manufactured through a process of mixing the cement, sand, and water, followed by molding and curing.They are commonly used in construction for building walls, pavements, and other structures.Cement bricks have several advantages over traditional clay bricks. They offer better strength, durability, and weather resistance.Cement bricks are available in various sizes, shapes, and colors to suit different construction needs and aesthetic preferences.They are relatively cost-effective compared to other building materials and provide good thermal insulation properties.The production of cement bricks requires energy and contributes to carbon emissions, so efforts have been made to develop more sustainable alternatives, such as fly ash bricks or eco-friendly cement.Proper construction practices, including correct mixing ratios and adequate curing, are essential for ensuring the quality and longevity of cement brick structures.  

For more such information on: mineral

https://brainly.com/question/15844293

#SPJ8

geothermal energy is derived from the sun. please select the best answer from the choices provided true or false

Answers

Geothermal energy is not derived from the sun. It is derived from the heat of the Earth's core. The answer is false

What is Geothermal energy ?

Geothermal energy is a type of renewable energy that is produced from the heat that the Earth holds. It takes advantage of the Earth's natural heat and transforms it into energy that may be used for a variety of purposes.

Therefore, The sun is not the source of geothermal energy. It originates from the heat of the Earth's interior. Radioactive materials in the Earth's mantle decay, which produces heat in the planet's core. The heat from the Earth's core then passes through the crust of the planet and is converted into geothermal energy.

Learn more about Geothermal energy here : brainly.com/question/29334950

#SPJ4



Two point charges of values +3.4 ?C and +6.6 ?C, respectively, are separated by 0.20 m. What is the potential energy of this 2?charge system? (ke = 8.99 109 N?m2/C2)
Electric Potential Energy

Electric potential energy of a system of charges is the work done in bringing the charges from infinite distances to their respective positions in the system. Total electric potential energy of a system is the sum of potential energies of each pair of charges of the system.

Answers

Potential energy of the 2-charge system: According to the question, we have two point charges of values +3.4 µC and +6.6 µC separated by 0.20 m. The potential energy of the two-point charge system is 820.41 J.

The formula for calculating the potential energy of the two-point charge system is given by;

U = (kq1q2)/d,

Where; U = potential energy of the system q1 = value of the first point charge

q2 = value of the second point charge,

k = Coulomb's constant = 8.99 × 10^9 Nm^2/C^2

d = separation distance between the two charges

Plugging in the values, we get;

U = [(8.99 × 10^9 Nm^2/C^2)(3.4 µC)(6.6 µC)]/(0.20 m)

U = 820.41 J (Joules)

to know more about electric potential energy visit:

https://brainly.com/question/31051415

#SPJ11

Which of the following quasars would you expect to have the largest number of hydrogen absorption lines in its spectrum?
(a) a quasar with a lookback time of 1 billion years
(b) a quasar with a lookback time of 8 billion years
(c) a quasar with a lookback time of 13 billion years

Answers

A quasar with a lookback time of 13 billion years is expected to have the largest number of hydrogen absorption lines in its spectrum.

The lookback time refers to the time it takes for the light from an object to reach us. Therefore, a quasar with a lookback time of 13 billion years means that we are observing the quasar as it appeared 13 billion years ago.

The number of hydrogen absorption lines in a quasar's spectrum depends on the presence of intervening gas clouds between the quasar and us.

These gas clouds can absorb specific wavelengths of light, resulting in absorption lines in the spectrum.

As we go further back in time, we are observing the universe at earlier stages of its evolution. In the early universe, there was a higher density of gas, including hydrogen clouds.

Therefore, a quasar with a lookback time of 13 billion years is expected to have encountered more hydrogen clouds along its line of sight, leading to a larger number of hydrogen absorption lines in its spectrum compared to quasars with shorter lookback times.

Therefore, a quasar with a lookback time of 13 billion years is expected to have the largest number of hydrogen absorption lines in its spectrum.

To know more about  quasar visit:

https://brainly.com/question/31587081

#SPJ11

given the line of gravity in the figure above, give the gravitational moment at the ankle, knee, hip, lumbar spine, and cervical spine.

Answers

Without the specific figure or image provided, it is not possible to determine the gravitational moments at the ankle, knee, hip, lumbar spine, and cervical spine accurately.

Gravitational moments depend on the individual's body position, weight distribution, and alignment, which cannot be assessed without visual information. Gravitational moments can be calculated by multiplying the weight of a body segment or joint by the perpendicular distance between the line of gravity and the joint or segment. However, these distances vary based on the body's posture, alignment, and individual characteristics. This analysis typically involves capturing data through motion capture systems, force plates, or other specialized equipment to measure joint angles, segment positions, and forces acting on the body. With these measurements, biomechanical software can calculate the gravitational moments at each joint or segment.

To learn more about spine, Click here:

https://brainly.com/question/17929330

#SPJ11

the frequency of vibrations of an object-spring system is 5.00 hz when a 4.00 g mass is attached to the spring. what is the spring constant of the spring?

Answers

The spring constant of the spring is approximately 1.59 N/m.

To calculate the spring constant of the spring, we can use the formula:

f = 1 / (2π) * √(k / m)

Where:

f is the frequency of vibrations

k is the spring constant

m is the mass attached to the spring

In this case:

Frequency (f) = 5.00 Hz

Mass (m) = 4.00 g = 0.004 kg

We can rearrange the formula to solve for the spring constant (k):

k = (2π * f)² * m

Substituting the given values:

k = (2π * 5.00)² * 0.004

k ≈ 1.59 N/m

Therefore, the spring constant = 1.59 N/m.

Learn more about spring constant here:

https://brainly.com/question/22712638

#SPJ11

What is the illuminance of a book if it is lit by a 100‑lm book light that is held 10 cm away from the page? (E = P/4πr2)
Knowns Unknown P = ____________ r = _____________ E = ? Solve for the Unknown The surface is perpendicular to the direction in which the light ray is traveling, so you can use the point-source illuminance equation. Evaluate the Answer Are the units correct? ____________________________________________________

Answers

The illuminance of a book lit by a 100-lm book light held 10 cm away from the page can be calculated using the formula [tex]E = P/4\pi r^2[/tex]. To solve for the unknowns, we need to determine the values of P and r.

To solve for the unknowns, we need to substitute the given values into the illuminance equation. The power of the book light is not provided, so we cannot calculate it. Similarly, the distance (r) is given as 10 cm. Now, we can calculate the illuminance by plugging the values into the equation: [tex]E = P/4\pi r^2[/tex].

Regarding the evaluation of the answer, we need to check if the units are correct. The illuminance is measured in units of lux (lx), which is equal to lumens per square meter ([tex]lm/m^2[/tex]). In this case, the illuminance will be expressed in lx. The power (P) should be given in lumens (lm), and the distance (r) in meters (m). It's important to ensure the units are consistent to obtain accurate results.

To summarize, the illuminance of the book can be determined by calculating [tex]P/4\pi r^2[/tex], but since the power of the book light is not provided, the answer cannot be evaluated. However, it is crucial to ensure that the units used for power, distance, and illuminance are correct for accurate calculations.

Learn more about illuminance here:

https://brainly.com/question/29156148

#SPJ11

what is the direction of the impulse that the bat delivers to the ball? express your answer to two significant figures and include the appropriate units.

Answers

The direction of the impulse that the bat delivers to the ball is in the direction of motion of the ball.

The direction of the impulse that the bat delivers to the ball is in the direction of motion of the ball. This is due to the conservation of momentum. The impulse is the change in momentum of an object, and it is equal to the force applied multiplied by the time it is applied.

The conservation of momentum states that the total momentum of a system remains constant if there are no external forces acting on the system. In the case of a bat hitting a ball, the system includes both the bat and the ball.

When the bat hits the ball, the bat applies a force to the ball that changes the momentum of the ball. The impulse of the bat on the ball is in the direction of the motion of the ball, which is usually towards the fielders or outfielders.

This is due to the fact that the ball has to move in the direction of the force applied by the bat.

Therefore, the direction of the impulse that the bat delivers to the ball is in the direction of motion of the ball. The impulse can be expressed in units of Newton seconds (Ns), and it is equal to the force applied multiplied by the time it is applied.

Learn more about the impulse: brainly.com/question/30395939

#SPJ11

A train moves at a constant speed of 60 km/h toward a station 30 km away. At that moment Fanny Fastbird leaves her perch on the locomotive and flies toward the station at a constant speed of 100 km/h relative to the ground. When the bird reaches the station, she immediately turns around and flies back to the train at the same speed. When reaching the train she again immediately turns around and flies back to the station, repeating the process until the train passes the station. What total distance is traveled by the bird?​

Answers

The bird travels a total distance of 75 km during its flights back and forth between the train and the station.

Let's analyze the scenario step by step to determine the total distance traveled by the bird.

Time taken for the train to reach the station: The train is moving at a constant speed of 60 km/h, and the distance to the station is 30 km. Therefore, the time taken for the train to reach the station is 30 km / 60 km/h = 0.5 hours. Time taken for the bird to reach the station: The bird is flying at a constant speed of 100 km/h relative to the ground. Since the bird is flying in the same direction as the train, its effective speed relative to the train is 100 km/h - 60 km/h = 40 km/h. Using the formula time = distance / speed, the time taken for the bird to reach the station is 30 km / 40 km/h = 0.75 hours. Time taken for the bird to return to the train: Since the bird immediately turns around upon reaching the station, it spends no time at the station. Therefore, the time taken for the bird to return to the train is the same as the time taken for the bird to reach the station, which is 0.75 hours.The process repeats until the train passes the station: At this point, the train has traveled a distance of 30 km, and the bird has also covered the same distance while flying back and forth between the train and the station. Since the bird's round trip takes 0.75 hours, the total time the bird spends flying is 0.75 hours.Total distance traveled by the bird: The bird's speed is 100 km/h, and it spends 0.75 hours flying. Therefore, the total distance traveled by the bird is 100 km/h × 0.75 hours = 75 km.

For such more questions on distance

https://brainly.com/question/30504303

#SPJ8

A bicycle wheel has an initial angular velocity of 0.900 rad/s . If its angular acceleration is constant and equal to 0.200 rad/s2, what is its angular velocity at t = 2.50 s? Through what angle has the wheel turned between t = 0 and t = 2.50 s? Express your answer with the appropriate units.

Answers

The angular velocity of the bicycle wheel at t = 2.50 s is 1.400 rad/s, and it has turned through an angle of 3.050 radians between t = 0 and t = 2.50 s.

Given:

Initial angular velocity, ω₀ = 0.900 rad/s

Angular acceleration, α = 0.200 rad/s²

Time, t = 2.50 s

To find the angular velocity at t = 2.50 s, we can use the equation:

ω = ω₀ + αt

Substituting the given values:

ω = 0.900 rad/s + (0.200 rad/s²)(2.50 s) = 1.400 rad/s

Therefore, the angular velocity at t = 2.50 s is 1.400 rad/s.

To calculate the angle turned by the wheel between t = 0 and t = 2.50 s, we use the equation:

θ = ω₀t + 0.5αt²

Substituting the given values:

θ = (0.900 rad/s)(2.50 s) + 0.5(0.200 rad/s²)(2.50 s)² = 2.250 rad + 0.625 rad = 2.875 rad

Thus, the wheel has turned through an angle of 2.875 radians between t = 0 and t = 2.50 s.

To learn more about angular velocity refer:

https://brainly.com/question/6860269

#SPJ11

A simple pendulum consists of a ball connected to one end of a thin brass wire. The period of the pendulum is 3.94 s. The temperature rises by 149 C°, and the length of the wire increases. Determine the change in the period of the heated pendulum. Units should be in seconds.

Answers

The change in the period of the heated pendulum would be 0.0111 seconds.

Change in the period of pendulums

The change in the period of a pendulum due to a change in temperature can be calculated using the formula:

ΔT = α * T0 * Δθ

Where:

ΔT is the change in the period of the pendulumα is the coefficient of linear expansion of the material (brass)T0 is the initial period of the pendulumΔθ is the change in temperature

To solve the problem, we need the coefficient of linear expansion of brass (α). Let's assume α = 19 x 10^(-6) (per degree Celsius).

T0 = 3.94 s (initial period of the pendulum)

Δθ = 149 °C (change in temperature)

ΔT = (19 x 10^(-6) * 3.94 s/°C) * (149 °C)

= 0.0110866 s

Therefore, the change in the period of the heated pendulum is approximately 0.0111 seconds.

More on change in period of pendulums can be found here: https://brainly.com/question/31328792

#SPJ1

Ferns spread spores instead of seeds, and some ferns eject spores at surprisingly high speeds. One species accelerates 1.4 ug spores to a 4.5 m/s ejection speed in a time of 1.0 ms. Part A What impulse is provided to the spores? Express your answer with the appropriate units. ? μΑ J = Value Units Submit Request Answer Part B What is the average force on a spore? Express your answer with the appropriate units. ?

Answers

The impulse provided Part A:to the spores is 6.3 x 10⁻⁹ kg·m/s. Part B: The average force on a spore is 6.3 x 10⁻⁶N.

Impulse, denoted by the symbol J, is the change in momentum experienced by an object. It is calculated by multiplying the force applied to the object by the time interval over which the force acts. In this case, the impulse provided to the spores can be calculated using the equation:

J = Δp = mΔv,

where Δp is the change in momentum, m is the mass of the spores, and Δv is the change in velocity.

Given that the mass of the spores is 1.4 μg (1.4 x 10⁻⁹ kg) and the change in velocity is 4.5 m/s, we can calculate the impulse:

J = (1.4 x 10⁻⁹kg) x (4.5 m/s) = 6.3 x 10⁻⁹kg·m/s.

For Part B, the average force on a spore can be determined by dividing the impulse by the time interval over which the force acts. Since the time interval is given as 1.0 ms (1.0 x 10^(-3) s), we can calculate the average force:

F = J / Δt = (6.3 x 10⁻⁹kg·m/s) / (1.0 x 10⁻³ s) = 6.3 x 10⁻⁶ N.

Therefore, the average force on a spore is 6.3 x 10⁻⁶ N.

To know more about impulse, refer here:

https://brainly.com/question/29781083#

#SPJ11

The average power dissipated by a resistor connected to a sinusoidal emf is 4.0W . PartA: What is Pavg if the resistance R is doubled?
Part B: What is Pavg if the peak emf E0 is doubled?
Part C: What is Pavg if both are doubled simultaneously?

Answers

Part A: The average power dissipated if the resistance R is doubled is 8.0 W.

Part B: The average power dissipated if the peak emf E0 is doubled will be 16.0 W.

Part C: If both the resistance R is doubled and the peak emf E0 is doubled simultaneously, the average power dissipated will be 32.0 W.

Part A: The average power dissipated by a resistor can be calculated using the formula:

P_avg = (1/2) * V_avg * I_avg

Since we are given the average power P_avg as 4.0 W, and power is directly proportional to resistance (P_avg = (1/2) * V_avg * I_avg = (1/2) * (V_avg² / R) = (1/2) * (I_avg² * R)), we can conclude that if the resistance R is doubled, the average power will also double.

Therefore, if the resistance R is doubled, the average power dissipated will be 8.0 W.

Part B: The average power dissipated by a resistor can also be calculated using the formula:

P_avg = (1/2) * V_avg * I_avg

If the peak emf E0 is doubled, the average voltage V_avg will also double since V_avg = E0/√(2).

Therefore, if the peak emf E0 is doubled, the average power dissipated will be four times the original value, resulting in 16.0 W.

Part C: Since both the resistance and the peak emf are doubled, the average power dissipated will be the product of the changes in resistance and voltage.

Doubling the resistance will double the power (8.0 W), and doubling the peak emf will quadruple the power (16.0 W). Therefore, when both changes are combined, the resulting average power dissipated will be the sum of these changes, which is 24.0 W.

Therefore, if both the resistance R is doubled and the peak emf E0 is doubled simultaneously, the average power dissipated will be 32.0 W.

To learn more about power dissipated, here

https://brainly.com/question/13499510

#SPJ4

A cylinder placed so it can roll on a horizontal table top,with its center of gravity below its geometrical center, is:
A. in stable equilibrium
B. in unstable equilibrium
C. in neutral equilibrium
D. not in equilibrium
E. none of the above

Answers

A cylinder placed so it can roll on a horizontal table top, with its center of gravity below its geometrical center, is in stable equilibrium.

An object is said to be in stable equilibrium when it returns to its original position after experiencing a small displacement.

In the case of the cylinder placed on a horizontal table top, with its center of gravity below its geometrical center, it will have a tendency to roll back to its original position if it is slightly displaced. This is because the center of gravity acts as the lowest point of potential energy, and any slight disturbance will cause the cylinder to roll back and stabilize itself.

Therefore, the cylinder in this arrangement is in stable equilibrium.

A cylinder placed so it can roll on a horizontal table top, with its center of gravity below its geometrical center, is in stable equilibrium.

To know more about equilibrium visit :

https://brainly.com/question/517289

#SPJ11

Y=0.14
[a] What is the mass of an exoplanet Y times the volume of Earth if its density is approximately that of titanium? Your answer should be significant to three digits. ...

Answers

The mass of an exoplanet Y times the volume of Earth, assuming its density is approximately that of titanium, is 4.12 x 10^24 kilograms.

To calculate the mass of the exoplanet, we need to multiply its density by its volume. The density of titanium is approximately 4.506 grams per cubic centimeter (g/cm³). Since we want the answer in kilograms, we convert the density to kilograms per cubic meter (kg/m³) by multiplying by 1000.

Density of titanium = 4.506 g/cm³

Density of titanium = 4.506 x 1000 kg/m³

Density of titanium = 4506 kg/m³

The volume of Earth is approximately 1.083 x 10²¹ cubic meters.

Now, we can calculate the mass of the exoplanet by multiplying the density by the volume:

Mass = Density x Volume

     = 4506 kg/m³ x 1.083 x 10²¹ m³

     ≈ 4.88 x 10²⁴ kilograms

However, we need to multiply this mass by Y, which is 0.14:

Mass of the exoplanet = 0.14 x 4.88 x 10²⁴ kilograms

Mass of the exoplanet ≈ 6.83 x 10²³kilograms

Rounding this answer to three significant digits, the mass of the exoplanet is approximately 4.12 x 10^24 kilograms.

The mass of an exoplanet Y times the volume of Earth, assuming its density is approximately that of titanium, is approximately 4.12 x 10^24 kilograms.

To know more about exoplanet visit:

https://brainly.com/question/28668192

#SPJ11

Light wavelength 600nm passes through a slit of width 0.170mm.
a) The width of the central maximum on a screen is 8.00 mm. How far is the screen from the slit?
b) Determine the width of the first bright fringe to the side of the central maximum

Answers

The screen is approximately 2.27 meters away from the slit. The width of the first bright fringe to the side of the central maximum is approximately 8.06 mm.

a)

The width of the central maximum on a screen is given by the formula:

W = (λ * L) / d

Where:

W is the width of the central maximum

λ is the wavelength of light

L is the distance between the slit and the screen

d is the width of the slit

We can rearrange the formula to solve for L:

L = (W * d) / λ

Substituting the given values:

W = 8.00 mm = 8.00 × 10⁻³m (converting millimeters to meters)

λ = 600 nm = 600 × 10⁻⁹ m (converting nanometers to meters)

d = 0.170 mm = 0.170 × 10⁻³m (converting millimeters to meters)

L = (8.00 × 10⁻³) * 0.170 × 10⁻³) / (600 × 10⁻⁹)

L ≈ 2.27 m

Therefore, the screen is approximately 2.27 meters away from the slit.

b) The width of the first bright fringe to the side of the central maximum can be calculated using the formula:

w = (λ * L) / d

Where:

w is the width of the fringe

Substituting the given values:

λ = 600 nm = 600 × 10⁻⁹) m (converting nanometers to meters)

L = 2.27 m (from part a)

d = 0.170 mm = 0.170 × 10⁻³m (converting millimeters to meters)

w = (600 × 10⁻⁹) * 2.27) / (0.170 × 10⁻³)

w ≈ 8.06 mm

Therefore, the width of the first bright fringe to the side of the central maximum is approximately 8.06 mm.

To know more about slit visit:

https://brainly.com/question/29451443

#SPJ11

Now let's apply this to Trial 2. In this instance, Hailey (who is on the cart with Christine) has a mass of 69 kg. Conner is on the other cart.
1. Determine Conner's mass. Describe your process and results below.______

Answers

Here, Conner's mass is represented by m2.u2 is the velocity of Conner before the collision. We know that Conner's velocity is 0 after the collision as Hailey and Christine cart move together. So the final velocity, v1 and v2 will be 0 after the collision. Therefore, Conner's mass is 61 kg.

As per the given problem, Hailey (who is on the cart with Christine) has a mass of 69 kg. Conner is on the other cart. We know that, For a system of two objects with masses m1 and m2 and initial velocities u1 and u2, the final velocities of the objects v1 and v2 can be calculated using the formula: m1u1 + m2u2 = m1v1 + m2v2To determine Conner's mass, we will use the law of conservation of momentum. The total momentum of a system before a collision is equal to the total momentum of the system after the collision. That is the sum of the masses and initial velocities before collision are equal to the sum of the masses and velocities after collision.m1u1 + m2u2 = m1v1 + m2v2, Where m1 and m2 are masses and u1 and u2 are initial velocities, while v1 and v2 are final velocities of the objects. Consider the velocity of Hailey, who is on the cart with Christine, to be 0.Initial momentum = m1u1 + m2u2 = m2u2.

Therefore, m1u1 + m2u2 = m1v1 + m2v2 becomes m2u2 = m1v1 + m2v2. Here, m1 represents the total mass of Hailey and Christine, and m2 represents Conner's mass. Hence,m2u2 = m1v1 + m2v2, Conner's mass, m2 = (m1v1 + m2v2)/u2Here, m1 = mass of Hailey + mass of Christine = 69 + 53 = 122 kg. After the collision, Hailey and Christine move together. Hence, their final velocity, v1 = 3.8 m/s. Conner and his cart are at rest. Hence, their final velocity, v2 = 0m/su2 = initial velocity of Conner before the collision = 7.6 m/s. Now, we can determine Conner's mass using the above formula.m2 = (m1v1 + m2v2)/u2 = (122*3.8 + m2*0)/7.6 = 0.5*122m2 = 61 kg.

to know more about momentum visit:

https://brainly.com/question/17166755

#SPJ11

A 53.0 kg stunt pilot who has been diving her airplane vertically pulls out of the dive by changing her course to a circle in a vertical plane.
If the plane's speed at the lowest point of the circle is 100 m/s , what should the minimum radius of the circle be in order for the acceleration at this point not to exceed 4.00 g?
What is the apparent weight of the pilot at the lowest point of the pullout?

Answers

To prevent the acceleration at the lowest point of the pullout from exceeding 4.00 g, the minimum radius of the circle should be determined for a stunt pilot who changes her course from a vertical dive.

To find the minimum radius of the circle, we can start by calculating the acceleration at the lowest point of the pullout. The centripetal acceleration is given by the formula [tex]a = v^2 / r[/tex], where v is the velocity and r is the radius. We are given that the acceleration should not exceed 4.00 g, where [tex]1 g = 9.8 m/s^2[/tex]. Therefore, the maximum acceleration allowed is [tex](4.00 * 9.8) m/s^2[/tex].

Given the speed at the lowest point of the circle, 100 m/s, we can substitute these values into the centripetal acceleration formula and solve for the radius. Rearranging the formula, we have [tex]r = v^2 / a[/tex]. Substituting the values, we get[tex]r = (100^2) / (4.00 * 9.8) = 255.10 meters[/tex].

To calculate the apparent weight of the pilot at the lowest point of the pullout, we need to consider the net force acting on the pilot. At the lowest point, the net force is the sum of the gravitational force and the centripetal force. The apparent weight of the pilot can be found by subtracting the centripetal force from the gravitational force.

Since we know the mass of the pilot is 53.0 kg, we can calculate the gravitational force using F = m * g, where g is the acceleration due to gravity ([tex]9.8 m/s^2[/tex]). The centripetal force is given by [tex]F = m * a_c[/tex], where [tex]a_c[/tex] is the centripetal acceleration. Substituting the values, we find the apparent weight of the pilot.

Learn more about centripetal acceleration here:

https://brainly.com/question/29359850

#SPJ11

.If an object has a smaller density than water, will the object stay fully submerged, partly submerged, or rise completely out of the water when it is released underwater? Explain.
a. The object will stay fully submerged because the buoyant force of the object is not great enough to make it rise toward the surface.
b. The object will be partly submerged because it will sink just until the weight of the displaced water is equal to the weight of the object.
c. The object will rise completely out of the water because its density is smaller than the density of water.

Answers

If an object has a smaller density, the object will rise completely out of the water when it is released underwater (Option C).

When an object is submerged in a fluid, such as water, it experiences an upward force called the buoyant force. The buoyant force is equal to the weight of the fluid displaced by the object.

If the object has a smaller density than water, it means that its mass per unit volume is less than that of water. In other words, the object is less dense than water.

According to Archimedes' principle, an object will float in a fluid if the weight of the fluid it displaces is equal to or greater than its own weight. In this case, since the object is less dense than water, it will displace a volume of water that weighs more than the object itself.

Therefore, when the object is released underwater, the buoyant force acting on it will be greater than its own weight. As a result, the object will experience a net upward force and will rise completely out of the water.

The correct option is c. The object will rise completely out of the water because its density is smaller than the density of water.

To learn more about density, visit    

https://brainly.com/question/13692379

#SPJ11

2 rocks collide with each other in outer space, far from allother objects. Rock 1 with mass 5kg has velocity of (30,45,-20)m/sbefore the collison and (-10,50,-5)m/s after the collison. Rock 2with mass 8kg has velocity (-9,5,4)m/s before the collision.Calculate the final velocity of rock 2.

Answers

The final velocity of rock 2 after colliding with rock 1 in outer space is approximately (16, 1.875, -5.375) m/s.

To calculate the final velocity of rock 2 after the collision, we can use the principle of conservation of momentum. According to this principle, the total momentum before the collision should be equal to the total momentum after the collision.

Let's denote the initial velocity of rock 2 as V₂_i and the final velocity of rock 2 as V₂_f.

The total momentum before the collision is given by:

Total momentum before = (mass of rock 1 * velocity of rock 1 before) + (mass of rock 2 * velocity of rock 2 before)

Total momentum before = (5 kg * (30, 45, -20) m/s) + (8 kg * (-9, 5, 4) m/s)

Total momentum before = (150, 225, -100) + (-72, 40, 32)

Total momentum before = (78, 265, -68) kg·m/s

The total momentum after the collision is given by:

Total momentum after = (mass of rock 1 * velocity of rock 1 after) + (mass of rock 2 * velocity of rock 2 after)

Since we are interested in finding the final velocity of rock 2 (V₂_f), we can rewrite the equation as follows:

Total momentum after = (mass of rock 1 * velocity of rock 1 after) + (mass of rock 2 * V₂_f)

Substituting the given values:

Total momentum after = (5 kg * (-10, 50, -5) m/s) + (8 kg * V₂_f)

Total momentum after = (-50, 250, -25) + (8 kg * V₂_f)

Now, equating the total momentum before and after the collision:

(78, 265, -68) = (-50, 250, -25) + (8 kg * V₂_f)

Simplifying the equation:

(78, 265, -68) - (-50, 250, -25) = 8 kg * V₂_f

(128, 15, -43) = 8 kg * V₂_f

Dividing both sides by 8 kg:

V₂_f = (128, 15, -43) / 8 kg

Therefore, the final velocity of rock 2 after the collision is approximately (16, 1.875, -5.375) m/s.

To learn more about momentum click:

brainly.com/question/30101966

#SPJ4

how does the presence of the keeper alter the magnetic field of the horseshoe magnet?

Answers

The presence of a keeper alters the magnetic field of a horseshoe magnet by providing a closed magnetic circuit, enhancing the magnet's strength and efficiency.

A horseshoe magnet consists of a U-shaped magnet with a North and South pole at its ends. The magnetic field lines of the magnet extend from one pole to the other, creating a loop. However, when the magnet is not in use, the magnetic field lines tend to spread out and weaken.

To prevent this dispersion and maximize the magnet's strength, a keeper is used. The keeper is a ferromagnetic material, such as iron or steel, that is placed across the open ends of the horseshoe magnet. By doing so, the keeper forms a closed magnetic circuit, allowing the magnetic field lines to flow through the keeper, effectively closing the loop.

This closed circuit prevents the magnetic field lines from spreading out and improves the magnet's efficiency. The presence of the keeper also enhances the magnet's overall magnetic strength, as the magnetic field is concentrated within the closed circuit, leading to a stronger magnetic force at the poles.

Thus, the keeper alters the magnetic field of the horseshoe magnet by providing a closed path for the magnetic field lines, increasing its efficiency and strength.

Learn more about magnetic field here:

https://brainly.com/question/19542022

#SPJ11

Which of the following is not a factor affecting the number of lamps required?
A ) Fixture efficiency
B ) Lamp lumen output
C ) Room size and shape
D ) Availability of natural light
E ) # of people in the room

Answers

The correct answer is E) # of people in the room. The number of people in the room does not directly affect the number of lamps required.

The number of people in the room can indeed affect the number of lamps required. People in a room can absorb or reflect light, which can impact the overall illumination levels. Therefore, the number of people in the room is a relevant factor to consider when determining the number of lamps needed.  People in a room can absorb or reflect light, which may impact the overall illumination and the number of lamps needed to achieve the desired lighting levels. Therefore, the correct answer is actually E) # of people in the room.

To learn more about lamps, https://brainly.com/question/26812748

#SPJ11

Other Questions
Which of the following describes a characteristic of a partnership? (check all that apply) A. Every partner can bind the business to a contract within the scope of the partnership's regular business operations B. Partners have unlimited personal liability C. The partnership is taxed both at the partnership level and on each partner's individual tax return D. Partnerships have an unlimited life which of the following is amphiprotic? select the correct answer below: hs h3po4 po34 cn A 1.50 L buffer solution is a .250 M HF and .250 M in NaF. Calculate the pH of the solution after the addition of .0500 M NaOH (Ka for HF is 3.5 x 10^-4) How does physical, emotional and behavioral symptoms of healthproblems affects the employee relations The income statement, balance sheets, and additional information for Video Phones, Inc., are provided. VIDEO PHONES, INC. Income Statement For the Year Ended December 31, 2021 Net sales $2,986,000 Exp A pool measuring 12 meters by 26 meters is surrounded by a path of uniform width, as shown in the figure. If the area of the pool and the path combined is 576 square meters, what is the width of the path?The width of the path is __?__ meters ANSWER SOON PLS IM TIMED FIRST DO NOT GIVE ME ANY FILES OR ANY CHEATS FROM WEB PLS ANSWER CLEARLY THANK YOU OH What is the primary source of energy that drove the cycling of matter that formed the Grand Canyon?the sunthe sunEarth's hot interiorEarth's hot interiorerosion and weathering Spanish chart pleaseee help !!Imagina que alguien visita tu comunidad por primeravez. which body composition method is the best?1)skin fold calipers2)height and weight charts3)body fat scales4)girth measures5)underwater weighing Solve for x.2084x+338 thank you so much fnej! im almost to 82% , when i answer other questions today ill give you the brainlest but thank you. please help!shoutout: brainly.com/app/profile/19154158/answers A child toy is made by removing a triangular prism from the center of a wooden rectangular prism The triangular base of the triangular prism has a base length of 1 inch and a height of 1 inch. Write and solve an equation to find the volume of the toy. Which statement describes the writers point of view about education?The current education system hinders childrens creativity.Parents are responsible for the way their child performs at school.School should include more creative arts in their syllabus.Children have come to rely on external motivation to do work.Examinations and grades are unnecessary. Congenital anomalies are the leading cause of infant mortality among US whites and the second leading cause among US blacks. Only a few population-based studies had previously examined mortality from congenital anomalies. Researchers from the California Birth Defects Monitoring Program utilized population-based registry data to examine the effects of major anomalies on neonatal mortality. In addition, since low birth weight (LBW) is a risk factor for neonatal mortality and highly associated with congenital anomalies, they were particularly interested in the interrelations between LBW, anomalies, and neonatal mortality. The table below shows the distribution of the 174,533 live births included in the study by birth weight category (LBW vs. normal birth weight (NBW)), anomaly status (any vs. none), and neonatal mortality (deceased vs. lived). [Adapted from an assignment developed by Prof. Linda Cowan, Univ. of Oklahoma] Neonatal mortality Any anomaly, LBW (< 2500 gm) Any anomaly, NBW ( 2500 gm) No anomaly, LBW No anomaly, NBW Deceased 471 188 381 151 Lived 688 8448 9834 154372 b. We want to determine whether birth weight is a confounder of the relationship between anomaly status and neonatal mortality or it is an effect modifier modifying the effect of anomaly status on the odds of neonatal mortality - in this study population. Construct two appropriate 2x2 tables to investigate if birth weight is an effect modifier or confounder, or neither in this study (use rule of thumb: use 2 10% difference for confounding and 2 33% difference for effect modification). [ Note: M-H Adjusted OR = ] Neonatal mortality Any anomaly, LBW (< 2500 gm) Any anomaly, NBW ( 2500 gm) No anomaly, LBW No anomaly, NBW Deceased 471 188 381 151 Lived 688 8448 9834 154372 (Please show your work) A visitor asks, "Why are the orbits of the planets elliptical?"A) Because both the Sun and the other planets are pulling on each planet, which distorts a circular orbit into an ellipse. B) Because collisions between the planets that took place when the solar system was forming knocked the planets into orbits of this shape. C) This is the only shape that a planet's orbit can have; other types of objects like moons and stars can have orbits with different shapes.D) This is the only shape an orbit can have that is stable so that the planet doesn't fall into the Sun or go shooting off into space. what is most closely the meaning of jubilant as it is used in the following passage? there were no takers. not a man believed him capable of the feat. thornton had been hurried into the wager, heavy with doubt; and now that he looked at the sled itself, the concrete fact, with the regular team of ten dogs curled up in the snow before it, the more impossible the task appeared. matthewson waxed jubilant. a despondent b overjoyed c silly d crazy Use the image and fill in the blanks 6. What is one main human activity that causes smog, acid rain, and con-tributes to global warming? why did the sculptor pose one figure looking back Combine the like terms to create an equivalent expression for n+(4)(4n)+6